Find limit using L'Hopital's rule, ln, and e: how did they do these steps?

Click For Summary
SUMMARY

The discussion focuses on applying L'Hopital's rule to evaluate the limit of the expression involving the natural exponential function and logarithms. Specifically, it examines the transition from the limit of an exponential function to the limit of its exponent, justified by the continuity of the exponential function. The key steps involve recognizing the indeterminate form \(1^\infty\) and using properties of logarithms to simplify the limit expression. The final goal is to compute \(\lim_{x\to\infty} e^{x\log(1+\frac{1}{x^2})}\) using L'Hopital's rule.

PREREQUISITES
  • Understanding of L'Hopital's rule
  • Familiarity with the properties of logarithms and exponents
  • Knowledge of limits and indeterminate forms
  • Basic calculus concepts related to continuity
NEXT STEPS
  • Study the application of L'Hopital's rule in various indeterminate forms
  • Learn about the continuity of exponential functions in limit evaluations
  • Explore the properties of logarithms in simplifying limit expressions
  • Practice solving limits involving exponential and logarithmic functions
USEFUL FOR

Students studying calculus, particularly those learning about limits, exponential functions, and L'Hopital's rule. This discussion is beneficial for anyone seeking to deepen their understanding of evaluating complex limits in mathematical analysis.

Nishiura_high
Messages
7
Reaction score
0

Homework Statement


This is from an online answer, and I don't understand the steps that it took. How did they go from the first red box to the second red box?

screenshot2.png



Homework Equations



L'Hopital's rule
Laws of exponents


The Attempt at a Solution


I am really really confused. It looks like they took what e was raised to, and found the limit of that... but what rule allowed them to "Finally, plug in the original limit"?
 
Physics news on Phys.org
So the step is

\lim_{x\rightarrow\infty} e^{x\log(1+1/x^2)}=\exp\left( \lim_{x\rightarrow\infty} \left( x\log\left( 1+\frac{1}{x^2} \right) \right) \right)

Which is justified by the continuity of the exponential. The general rule is for f continuous

\lim_{x\rightarrow\infty} f(g(x))=f \left( \lim_{x\rightarrow\infty} g(x) \right)

This is sometimes called the composition rule for limits.

Then to keep out expression from being messy we can substitute
call the limit a, so we want to find e^a
we can find a first, then find e^a
 
Last edited:
First, you need to determine whether or not this is an indeterminate form. As x tends to infinity, the base comes closer to one, and the exponent naturally increases to infinity. Since 1^\infty is an indeterminate form, you can use L'Hopital's rule here. The first step is to use the property e^{ln(x)}=x This is a very useful property because the logarithm will turn this limit into an indeterminate product, which is much easier to work with. So back to the problem:
\lim_{x\to\infty} e^{ln((1+\frac{1}{x^2})^x)}
\lim_{x\to\infty} e^{xln(1+\frac{1}{x^2})}
Once again, we have a product of two infinities, so this is another indeterminate form. We can rewrite this:
\lim_{x\to\infty} e^\frac{ln(1+\frac{1}{x^2})}{\frac{1}{x}}
This looks pretty nasty, but it's doable. Can you use L'Hopital's rule now?
 
Last edited:
Question: A clock's minute hand has length 4 and its hour hand has length 3. What is the distance between the tips at the moment when it is increasing most rapidly?(Putnam Exam Question) Answer: Making assumption that both the hands moves at constant angular velocities, the answer is ## \sqrt{7} .## But don't you think this assumption is somewhat doubtful and wrong?

Similar threads

Replies
5
Views
2K
  • · Replies 5 ·
Replies
5
Views
2K
  • · Replies 21 ·
Replies
21
Views
7K
  • · Replies 1 ·
Replies
1
Views
3K
  • · Replies 10 ·
Replies
10
Views
2K
  • · Replies 12 ·
Replies
12
Views
2K
  • · Replies 4 ·
Replies
4
Views
2K
  • · Replies 3 ·
Replies
3
Views
2K
Replies
12
Views
2K
  • · Replies 9 ·
Replies
9
Views
2K